2021年入試問題研究に戻る

特色入試総人理系2番解答

(1)  $ n\geqq 1 $ に対して, $ \displaystyle f_n(x)=e^x-\left(1+\sum_{k=1}^n\dfrac{x^k}{k!}\right) $ とおく。
このとき, $ \displaystyle f_n'(x)=f_{n-1}(x) $ となる。 また, $ f_n(0)=1-1=0 $ である。
$ x\geqq 0 $ で $ f_n(x)\geqq 0 $ であることを, $ n $ に関する数学的帰納法で示す。
$ n=1 $ のとき。 $ f_1(x)=e^x-(1+x) $ で, $ f_1'(x)=e^x-1 $ である。 $ x\geqq 0 $ より $ e^x\geqq 1 $ なので, $ f_1'(x)\geqq 0 $ である。 かつ $ f_1(0)=0 $ なので, $ x\geqq 0 $ において $ f_1(x)\geqq 0 $ である。
$ n=k $ のとき,成立するとする。
$ f'_{k+1}(x)=f_k(x)\geqq 0 $ ,かつ $ f_{k+1}(0)=0 $ より, $ x\geqq 0 $ で $ f_{k+1}(x)\geqq 0 $ である。
これよりすべての自然数 $ n $ に対して, $ x \geqq 0 $ において $ f_n(x)\geqq 0 $ ,つまり $ \displaystyle e^x\geqq 1+\sum_{k=1}^n\dfrac{x^k}{k!} $ となる。

(2)  $ C $ 上の点 $ (t,\ e^t) $ での $ C $ の接線は $ y=e^t(x-t)+e^t $ である。これが原点を通るのは, $ t=1 $ のときで, このとき接線は $ y=ex $ である。
つまり, $ a=e $ のとき, $ y=e^x $ と $ y=ex $ は点 $ (1,\ e) $ で接する。 $ y=e^x $ のグラフは, $ y''=e^x>0 $ より下に凸である。 よって, $ l_a $ と $ C $ がちょうど2点で交わるような $ a $ の範囲は $ a>e $ である。

(3)  $ x_1(a) $ と $ x_2(a) $ は $ ax=e^x $ を満たす。 \begin{eqnarray*} S(a)&=&\int_{x_1(a)}^{x_2(a)}\left(ax-e^x \right)\,dx =\left[\dfrac{a}{2}x^2-e^x \right]_{x_1(a)}^{x_2(a)}\\ &=&\dfrac{a}{2}\left\{x_2^2(a)-x_1^2(a) \right\}-\left\{e^{x_2(a)}-e^{x_1(a)} \right\}\\ &=&\dfrac{a}{2}\left\{x_2^2(a)-x_1^2(a) \right\}-a\left\{x_2(a)-x_1(a)\right\}\\ T(a)&=&\int_{x_1(a)}^{x_2(a)}e^x \,dx=e^{x_2(a)}-e^{x_1(a)}\\ &=&a\left\{x_2(a)-x_1(a)\right\} \end{eqnarray*} よって, \[ \dfrac{S(a)}{T(a)}=\dfrac{1}{2}\left\{x_2(a)+x_1(a) \right\}-1 \] であり,これより \[ \dfrac{1}{\log a}\cdot\dfrac{S(a)}{T(a)}=\dfrac{x_2(a)+x_1(a)}{2\log a}-\dfrac{1}{\log a} \] となる。
$ a\to \infty $ のとき, $ \dfrac{1}{\log a}\to 0 $ であり,かつ直線 $ l_a $ は $ y $ 軸に近づく。よって, $ x_1(a)\to 0 $ , $ x_2(a)\to \infty $ である。 ここで, $ X=x_2(a) $ とおく。 $ aX=e^X $ より, \[ \log a+\log X=X \] である。よって, \[ \dfrac{x_2(a)}{2\log a}=\dfrac{X}{2(X-\log X)} \] となる。これより \[ \lim_{a \to \infty}\dfrac{x_2(a)}{2\log a}=\lim_{X \to \infty}\dfrac{X}{2(X-\log X)} =\dfrac{1}{2}\cdot \lim_{X \to \infty}\dfrac{1}{1-\dfrac{\log X}{X}} \] ここで, $ \log X=t $ とおくと,1より大きい $ X $ に対して $ t>0 $ であり, (1)より, $ e^t\geqq t+\dfrac{1}{2}t^2 $ なので \[ \dfrac{\log X}{X}=\dfrac{t}{e^t} \leqq \dfrac{t}{t+\dfrac{1}{2}t^2} =\dfrac{1}{1+\dfrac{1}{2}t} \] であるから \[ 0\leqq \lim_{X \to \infty}\dfrac{\log X}{X}\leqq \lim_{t \to \infty}\dfrac{1}{1+\dfrac{1}{2}t}=0 \] となる。 したがって, \[ \lim_{a \to \infty}\dfrac{1}{\log a}\cdot\dfrac{S(a)}{T(a)} =\dfrac{1}{2} \] である。

問題